K
Khách

Hãy nhập câu hỏi của bạn vào đây, nếu là tài khoản VIP, bạn sẽ được ưu tiên trả lời.

Xét \(n=2^k.q\) trong đó \(q\)là số lẻ

ta có \(2^n+1=\left(2^{2^k}\right)^q+1⋮\left(2^{2^k}+1\right)\)

vì \(q\)lẻ

ta được:

nếu \(k\ge1\) thì là hợp số

\(k=0\) cũng là hợp số

nên \(q=1\)

khi đó \(n=2^k\left(đpcm\right)\)

6 tháng 8 2020

Tại sao mà  \(k\ge1\)lại suy ra q = 1

6 tháng 8 2020

\(A=\left(2+1\right)\left(2^2+1\right)\left(2^4+1\right)\left(2^8+1\right)\)

\(=\left(2-1\right)\left(2+1\right)\left(2^2+1\right)\left(2^4+1\right)\left(2^8+1\right)\)

\(=\left(2^2-1\right)\left(2^2+1\right)\left(2^4+1\right)\left(2^8+1\right)\)

\(=\left(2^4-1\right)\left(2^4+1\right)\left(2^8+1\right)\)

\(=\left(2^8-1\right)\left(2^8+1\right)=2^{16}-1\)

\(B=\left(3x-1\right)^2+\left(5-3x\right)^2+\left(6x-2\right)\left(5-3x\right)\)

\(=\left(3x-1\right)^2+\left(5-3x\right)^2+2.\left(3x-1\right)\left(5-3x\right)\)

\(=\left(3x-1+5-3x\right)^2=4^2=16\)

6 tháng 8 2020

Bài làm:

Ta có: \(5\left(m-3\right)^2-5\)

\(\ge-5\left(\forall m\right)\)

Dấu "=" xảy ra khi: \(\left(m-3\right)^2=0\Rightarrow m=3\)

Vậy \(Min=-5\Leftrightarrow m=3\)

6 tháng 8 2020

\(5\left(m-3\right)^2-5\)

Ta có : \(5\left(m-3\right)^2\ge0\forall m\Rightarrow5\left(m-3\right)^2-5\ge-5\)

Dấu " = " xảy ra <=> m - 3 = 0 => m = 3

Vậy GTNN của biểu thức = -5, đạt được khi m = 3

Bài 1

Cách giải

Gọi số thứ nhất là x (x nguyên dương; x < 120) 

Thì số thứ hai là 3x

Vì Tổng của chúng bằng 120 nên ta có phương trình: x + 3x = 120 <=> x = 30 (Thỏa mãn điều kiện đặt ẩn) Vậy số thứ nhất là 30, số thứ hai là 90.

Bài 2

Thể tích của hình lăng trụ đứng tam giác là: V = S.h = 1/2.3.4.7 = 42(cm3)

Hok tốt ^^

6 tháng 8 2020

1/ Gọi số bé là x ( x thuộc N* , x < 120 )

=> Số lớn = 120 - x

Theo đề bài ta có : 120 - x = 3x

                         <=> 120 = 3x + x

                         <=> 120 = 4x

                         <=> x = 30 ( tmđk )

Vậy số bé = 30

       số lớn = 120 - 30 = 90

2/ Thể tích hình lăng trụ đứng = 1/2(3.4).7 = 42cm3

6 tháng 8 2020

Ta có: \(\frac{x^2}{1+2yz}+\frac{y^2}{1+2zx}+\frac{z^2}{1+2xy}\)

\(\ge\frac{\left(x+y+z\right)^2}{3+2\left(xy+yz+zx\right)}\ge\frac{\left(x+y+z\right)^2}{3+2\left(x^2+y^2+z^2\right)}\)

\(=\frac{\left(x+y+z\right)^2}{3+2}=\frac{\left(x+y+z\right)^2}{5}\)

Mà \(\left(x+y+z\right)^2\le3\left(x^2+y^2+z^2\right)=3\)

Nên thay vào ngược dấu

=> ch bt lm

6 tháng 8 2020

Nói chung khá đơn giản. Em chứng minh bất đẳng thức sau đây là được.

\(\frac{x^2}{1+2yz}=\frac{x^2}{x^2+\left(y^2+z^2+2yz\right)}=\frac{x^2}{x^2+\left(y+z\right)^2}\ge\frac{1}{25}\cdot\frac{17x^2-y^2-z^2}{x^2+y^2+z^2}\)

Có thể chứng minnh nó bằng cách: \(f\left(x,y,z\right)=\frac{x^2}{x^2+\left(y+z\right)^2}-\frac{1}{25}\cdot\frac{17x^2-y^2-z^2}{x^2+y^2+z^2}\)

Ta chứng minhL \(f\left(x,y,z\right)\ge f\left(x,\frac{y+z}{2},\frac{y+z}{2}\right)\ge0\) (quy đồng phát là ra nhân tử (y-z)^2 nên hiển nhiên:v)

Tương tự cộng lại. Xong.

Cách Cauchy-SChwarz:

Chứng minh theo trình tự: \(\Sigma\frac{x^2}{x^2+\left(y+z\right)^2}\ge\frac{\left(x^2+y^2+z^2\right)^2}{\Sigma x^2\left[x^2+\left(y+z\right)^2\right]}\ge\frac{3}{5}\)

6 tháng 8 2020

Không, thật ra với mọi k \(\inℕ^∗\)thì ak - 1 \(⋮\)a - 1

Bg

Ta có ak - 1 (a, k \(\inℕ^∗\))

=> ak - 1 = ak - ak - 1 + ak - 1 - ak - 2 +...+ ak - 1

=> ak - 1 = (ak - ak - 1) + (ak - 1 - ak - 2) +...+ (ak - 1)

=> ak - 1 = [(ak - 1(a - 1)] + [ak - 2(a - 1)] +...+ 1(ak - 1)

=> ak - 1 = (a - 1)(ak - 1 + ak - 2 +...+ 1) \(⋮\)a - 1

=> ak - 1 \(⋮\)a - 1

=> ĐPCM

Bài 1

Gọi số học sinh lớp 8A là x (học sinh) ĐK: x ∈ N* và x < 80 

Số học sinh lớp 8B là 80 - x(học sinh) 

Số sách lớp 8A ủng hộ là 2x (quyển) 

Số sách lớp 8B ủng hộ là 3(80 - x) (quyển) 

Theo bài ta có phương trình: 

<=>2x + 3(80 - x) = 198 

<=>2x + 248 - 3x = 198 

x = 42 (thoả mãn điều kiện) Vậy số học sinh lớp 8A là 42 học sinh,số học sinh lớp 8B là 38 học sinh.

Bài 2

Gọi độ dài quãng đường AB là x (km)      (ĐK: x > 0) 

Thời gian lúc đi là: x/35 (giờ), thời gian lúc về là : x/42 (giờ).

Theo bài ra ta có phương trình: x/35 - x/42 = 1/2

Giải phương trình được x = 105, thoả mãn điều kiện của ẩn. Trả lời : Vậy độ dài quãng đường AB là 105 km.

Hok tốt ^^

Bài 1: Gọi x (h/s) là số h/s của lớp 8A (0 < x < 80 ). Số h/s của lớp 8D là: 80 - x

Số cách lớp 8a ủng hộ là 2x (quyển); số sách lớp 8D ủng hộ là 3(80 - x) (quyển)

Theo đề bài 2 lớp góp đc 198 nên ta có phương trình: 2x +3(80 - x) = 198 

<=> 2x + 240 - 3x = 198 => x = 42 (h/s) (TMĐK) => Số h/s lớp 8A là: 42 h/s 

Số h/s lớp 8D là: 80 - x = 80 - 24 = 56 (h/s) 

Bài 2: Gọi t(h) là thời gian đi (t > 0,5) - quãng đường AB (tính theo lúc đi) 35t 

                                                            - quãng đường AB (tính theo lúc về) 42(t - 0,5) 

Ta có phương trình: 35t = 42(t - 0,5) giải phương trình: 35t = 42(t-0,5) 

                                                                                        <=> 35t = 42t - 21 <=> -7t = -21 <=> t = 3

=> Quãng đường AB dài là: 35.3 = 105 (km)

6 tháng 8 2020

Có : \(\left(x+2y\right)^2+\left(2x-y\right)^2-5.\left(x+y\right)\left(x-y\right)-10\left(y+3\right)\left(y-3\right)\)

\(=x^2+4y^2+4xy-4xy+4x^2+y^2-5.\left(x^2-y^2\right)-10.\left(y^2-9\right)\)

\(=5x^2+5y^2-5x^2+5y^2-10y^2+90\)

\(=90\) không phụ thuộc vào giá trị biến.

6 tháng 8 2020

( x + 2y )2 + ( 2x - y )2 - 5( x + y )( x - y ) - 10( y + 3 )( y - 3 )

= x2 + 4xy + 4y2 + 4x2 - 4xy + y2 - 5( x2 - y2 ) - 10( y2 - 32 )

= x2 + 4xy + 4y2 + 4x2 - 4xy + y2 - 5x2 + 5y2 - 10y2 + 90

= 90 

Vậy giá trị của biểu thức không phụ thuộc vào biến ( đpcm )

6 tháng 8 2020

Đề thiếu với \(k=2n+1\) ( số lẻ )

Ta luôn có HĐT : \(x^{2n+1}+1^{2n+1}\)

\(=x^{2n+1}+1=\left(x+1\right)\left(x^{2n}-x^{2n-1}+x^{2n-2}-....+1\right)\)

Do đó : \(x^{2n+1}+1⋮x+1\)

Áp dụng vào bài toán thì : \(25^k+1⋮25+1\) với k lẻ.

6 tháng 8 2020

Bạn thử k = 2 thì đâu có được, sửa đề: CMR: 25k - 1 \(⋮\)25 - 1

Bg

Ta có: 25k - 1  (k \(\inℕ\))

=> 25k - 1 = 25k - 25k - 1 + 25k - 1 - 25k - 2 + 25k - 2 - 25k - 3 +...+ 25 - 1

=> 25k - 1 = (25k - 25k - 1) + (25k - 1 - 25k - 2) + (25k - 2 - 25k - 3) +...+ (25 - 1)

=> 25k - 1 = (25k - 1.25 - 25k - 1) + (25k - 2.25 - 25k - 2) + (25k - 3.25 - 25k - 3) +...+ 1.(25 - 1)

=> 25k - 1 = [25k - 1.(25 - 1)] + [25k - 2.(25 - 1)] + [25k - 3.(25 - 1)] +...+ 1.(25 - 1)

=> 25k - 1 = (25 - 1)(25k - 1 + 25k - 2 + 25k - 3 +...+ 1) \(⋮\)25 - 1

=> 25k - 1 \(⋮\)25 - 1

=> ĐPCM